LSAT and Law School Admissions Forum

Get expert LSAT preparation and law school admissions advice from PowerScore Test Preparation.

 Administrator
PowerScore Staff
  • PowerScore Staff
  • Posts: 8916
  • Joined: Feb 02, 2011
|
#59619
Complete Question Explanation
(The complete setup for this game can be found here: lsat/viewtopic.php?t=26510)

The correct answer choice is (D)

As with any List question, simply apply the rules to the answer choices. Remember to apply the rules in order of the easiest to “see” within the answers. In this game, that order would be rule #3, rule #2, and then rule #1.

Answer choice (A): This answer is eliminated by the first rule. In this instance, there is only one group separating P and M.

Answer choice (B): This answer choice is eliminated by the third rule. In this answer there is no group between F and V.

Answer choice (C): This answer is eliminated by the third rule because G marches in group 4.

Answer choice (D): This is the correct answer choice.

Answer choice (E): This answer violates both the first and second rules. Interestingly, if you misdiagram the first two rules by reversing the order of the variables (for example, “V __ F”), this answer would appear correct.

One of the great benefits of this question is that we are given a free hypothetical solution to the game, in this case J-P-G-F-M-V. This could be useful in a later question.

Get the most out of your LSAT Prep Plus subscription.

Analyze and track your performance with our Testing and Analytics Package.